Shocked Face

Geometry Level 5

Let R R be the region in the first quadrant outside of the circles x 2 + y 2 = 4 x^{2} + y^{2} = 4 and x 2 + ( y 3 ) 2 = 1 x^{2} + (y - 3)^{2} = 1 but inside the circle x 2 + ( y 1 ) 2 = 9. x^{2} + (y - 1)^{2} = 9.

The largest circle that can be inscribed in R R has radius a b \dfrac{a}{b} , where a a and b b are positive coprime integers. Find a + b a + b .


The answer is 13.

This section requires Javascript.
You are seeing this because something didn't load right. We suggest you, (a) try refreshing the page, (b) enabling javascript if it is disabled on your browser and, finally, (c) loading the non-javascript version of this page . We're sorry about the hassle.

4 solutions

Ujjwal Rane
Feb 10, 2016

If r r is the radius we seek, then A B = 1 ; B C = 2 , A D = r + 2 ; B D = 3 r ; C D = r + 1 AB = 1; BC = 2, AD = r+2; BD = 3-r; CD = r+1

Stewart's theorem gives 3 × 2 × 1 + ( 3 r ) 2 × 3 = ( r + 1 ) 2 × 1 + ( r + 2 ) 2 × 2 3 \times 2 \times 1 + (3-r)^2 \times 3 = (r+1)^2 \times 1+ (r+2)^2 \times 2 6 + 27 18 r = 2 r + 1 + 8 r + 8 6 + 27 -18r = 2r + 1 + 8r + 8 24 = 28 r 24 = 28r giving r = 6 7 r = \frac{6}{7} .

@Ujjwal Rane , we really liked your comment, and have converted it into a solution. If you subscribe to this solution, you will receive notifications about future comments.

Brilliant Mathematics Staff - 5 years, 4 months ago

Log in to reply

Thank you! Really appreciate that!

Ujjwal Rane - 5 years, 4 months ago

Let A A be the center of the circle x 2 + y 2 = 4 x^{2} + y^{2} = 4 , B B be the center of the circle x 2 + ( y 1 ) 2 = 9 x^{2} + (y - 1)^{2} = 9 , C C be the center of the circle x 2 + ( y 3 ) 2 = 1 x^{2} + (y - 3)^{2} = 1 and D D be the center of the sought after inscribed circle. Also, let r r be the radius of the circle with center D . D. Then, using the Cosine rule on Δ A C D \Delta ACD we have that

A D 2 = A C 2 + C D 2 2 A C C D cos ( A C D ) |AD|^{2} = |AC|^{2} + |CD|^{2} - 2*|AC|*|CD|*\cos(\angle ACD)

( 2 + r ) 2 = 9 + ( 1 + r ) 2 6 ( 1 + r ) cos ( A C D ) \Longrightarrow (2 + r)^{2} = 9 + (1 + r)^{2} - 6(1 + r)\cos(\angle ACD)

cos ( A C D ) = 3 r 3 ( 1 + r ) . \Longrightarrow \cos(\angle ACD) = \dfrac{3 - r}{3(1 + r)}.

Next, using the Cosine rule on Δ B C D \Delta BCD and noting that B C D = A C D \angle BCD = \angle ACD , we have that

B D 2 = B C 2 + C D 2 2 B C C D cos ( A C D ) |BD|^{2} = |BC|^{2} + |CD|^{2} - 2*|BC|*|CD|*\cos(\angle ACD)

( 3 r ) 2 = 4 + ( 1 + r ) 2 4 ( 1 + r ) cos ( A C D ) \Longrightarrow (3 - r)^{2} = 4 + (1 + r)^{2} - 4(1 + r)\cos(\angle ACD)

cos ( A C D ) = 2 r 1 1 + r . \Longrightarrow \cos(\angle ACD) = \dfrac{2r - 1}{1 + r}.

Equating the two expressions for cos ( A C D ) \cos(\angle ACD) gives us

3 r 3 ( 1 + r ) = 2 r 1 1 + r 3 r = 6 r 3 7 r = 6 r = 6 7 . \dfrac{3 - r}{3(1 + r)} = \dfrac{2r - 1}{1 + r} \Longrightarrow 3 - r = 6r - 3 \Longrightarrow 7r = 6 \Longrightarrow r = \dfrac{6}{7}.

Thus a + b = 6 + 7 = 13 . a + b = 6 + 7 = \boxed{13}.

I solved it and got a 'Correct!' so looked up the solution and got a 'Not answered' status. Anyway, could we use Stewart's Theorem on triangle ACD here?

If r r is the radius we seek, then A B = 1 ; B C = 2 , A D = r + 2 ; B D = 3 r ; C D = r + 1 AB = 1; BC = 2, AD = r+2; BD = 3-r; CD = r+1

Stewart's theorem gives 3 × 2 × 1 + ( 3 r ) 2 × 3 = ( r + 1 ) 2 × 1 + ( r + 2 ) 2 × 2 3 \times 2 \times 1 + (3-r)^2 \times 3 = (r+1)^2 \times 1+ (r+2)^2 \times 2 6 + 27 18 r = 2 r + 1 + 8 r + 8 6 + 27 -18r = 2r + 1 + 8r + 8 24 = 28 r 24 = 28r giving r = 6 7 r = \frac{6}{7} .

Ujjwal Rane - 6 years, 2 months ago

Log in to reply

Yes, Stewart's Theorem is perfect for this problem. You've mentioned this theorem before, and now that I've seen your application of it I will be more likely to make use of it in the future. :)

P.S.. I hope that the 'Not answered' status has corrected itself. If not, then it might be worth mentioning to Calvin in case there is a glitch in the system.

Brian Charlesworth - 6 years, 2 months ago

Log in to reply

Yes Brian, Stewart's theorem is pretty handy and I owe it to Brilliant! (they don't teach that in schools :-( I will let Calvin know about a possible bug or possibly a (connectivity + browser) state that leads to it.

Ujjwal Rane - 6 years, 2 months ago

I think you should probably make a note on how to solve these 'types' of problems because I don't think this problem is as hard as rating suggests(no offence sir) probably due to less number of solvers and high number of viewers, its just a suggestion and observation from my side :)

Krishna Sharma - 6 years, 2 months ago

Log in to reply

The solver-to-views ratio is fairly high, so I'm not sure why the rating is as high as it is. I thought I assigned it a level 4 rating when I posted the question, but it was late and I may have chosen level 5 by mistake. Once there are more views and solvers I think that the rating will come down, (it should probably be worth about 175 points rather than the present 275 points; does that sound about right?). I'm not sure what you mean by "make a note on how to solve these 'types' of problems". When I make these "fit the circle" types of problems I try to post a slightly different solution method for each, and for past problems others have posted alternate methods as well. I suppose that I could form a set of all the problems of this type that I have posted.

I also had in mind a more difficult problem, which involved finding the radii of the sequence of circles as we move clockwise around the x 2 + y 2 = 4 x^{2} + y^{2} = 4 circle within region R R , (extended down to the 4th quadrant). I wasn't able to find a simple strategy to do this last night, so I just posted this problem instead.

Brian Charlesworth - 6 years, 2 months ago

Log in to reply

Yes, I have seen that your solutions are always different but I solved similarly as yash,

  • Distance between centre equals sum of radii for externally tangent

  • Distance between centres equals difference of radii when internally tangent

Most of the problems I have solved with these 2 equations only

Hey but I remember that you have posted similar problem on sequence of circles around a circle while radius decrease harmonically(I couldn't solve that -_-) and about the points under 200 is okay :)

Krishna Sharma - 6 years, 2 months ago

Log in to reply

@Krishna Sharma Yes, that is an effective method. I'll have to think up a "circle" problem that can't be solved using these two steps and then dedicate it to you. :) The harmonically decreasing circle problem would qualify, (it was one of my favorites), so I'll follow through on the more difficult problem I proposed above to see if will qualify as well.

As for the point level for this problem, it's still going up for some reason. :P Not much I can do about that now.

Brian Charlesworth - 6 years, 2 months ago
Yash Choudhary
Mar 26, 2015

Let r r be the radius of circle C4.

C4 touches C1 internally, 3 r = h 2 + ( k 1 ) 2 r 2 6 r + 2 k + 8 = h 2 + k 2 1 \Rightarrow \quad 3-r=\sqrt { { h }^{ 2 }+{ (k-1) }^{ 2 } } \\ \Rightarrow \quad { r }^{ 2 }-6r+2k+8={ h }^{ 2 }+{ k }^{ 2 }\longrightarrow \boxed { 1 }

C4 touches C2 externally, 1 + r = h 2 + ( k 3 ) 2 r 2 + 2 r + 6 k 8 = h 2 + k 2 2 \Rightarrow \quad 1+r=\sqrt { { h }^{ 2 }+{ (k-3) }^{ 2 } } \\ \Rightarrow \quad { r }^{ 2 }+2r+6k-8={ h }^{ 2 }+{ k }^{ 2 }\longrightarrow \boxed { 2 }

C4 touches C3 externally, 2 + r = h 2 + k 2 r 2 + 4 r + 4 = h 2 + k 2 3 \Rightarrow \quad 2+r=\sqrt { { h }^{ 2 }+{ k }^{ 2 } } \\ \Rightarrow \quad { r }^{ 2 }+4r+4={ h }^{ 2 }+{ k }^{ 2 }\longrightarrow \boxed { 3 }

Solving equations 1. 2 and 3, we get r = 6 7 r=\frac { 6 }{ 7 } ,

Hence, answer is 6 + 7 = 13 . 6+7=\boxed { 13 } .

Did the same. +1

Aditya Tiwari - 6 years, 2 months ago

Using Descartes circle theorem . Let r 0 r \neq 0 be the radius we are seeking, then 2 ( 1 4 + 1 9 + 1 + 1 r 2 ) = ( 1 2 + 1 1 3 + 1 r ) 2 ( 1 r 7 6 ) 2 = 0 r = 6 7 . 2(\frac{1}{4} + \frac{1}{9} + 1 + \frac{1}{r^2}) = (\frac{1}{2} + 1 - \frac{1}{3} + \frac{1}{r})^2 \iff (\frac{1}{r} - \frac{7}{6})^2 = 0 \iff \boxed{r = \frac{6}{7}}. Or in otrher words,

Proposition.- Let R \mathbb{R} be the set of real numbers, then are equivalents:

a) r = 6 7 r = \frac{6}{7}

b) 1 r = 7 6 \frac{1}{r} = \frac{7}{6}

c) ( 1 r 7 6 ) 2 = 0 (\frac{1}{r} - \frac{7}{6})^2 = 0

d) 2 ( 1 4 + 1 9 + 1 + 1 r 2 ) = ( 1 2 + 1 1 3 + 1 r ) 2 2(\frac{1}{4} + \frac{1}{9} + 1 + \frac{1}{r^2}) = (\frac{1}{2} + 1 - \frac{1}{3} + \frac{1}{r})^2

Proof.-

a ) b ) a) \iff b) Trivial.

b ) c ) b) \Rightarrow c) If 1 r = 7 6 1 r 7 6 = 0 ( 1 r 7 6 ) 2 = 0 \frac{1}{r} = \frac{7}{6} \Rightarrow \frac{1}{r} - \frac{7}{6} = 0 \Rightarrow (\frac{1}{r} - \frac{7}{6})^2 = 0

c ) b ) c) \Rightarrow b) If ( 1 r 7 6 ) 2 = 0 ( 1 r 7 6 ) ( 1 r 7 6 ) = 0 1 r = 7 6 (\frac{1}{r} - \frac{7}{6})^2 = 0 \Rightarrow (\frac{1}{r} - \frac{7}{6}) \cdot (\frac{1}{r} - \frac{7}{6}) = 0 \Rightarrow \frac{1}{r} = \frac{7}{6} .

Hence, so far, we have already proved a ) b ) c ) a) \iff b) \iff c) . Now

a ) d ) a) \Rightarrow d) Trivial (calculation)

d ) c ) d) \Rightarrow c) If 2 ( 1 4 + 1 9 + 1 + 1 r 2 ) = ( 1 2 + 1 1 3 + 1 r ) 2 49 18 + 2 1 r 2 = ( 7 6 + 1 r ) 2 1 r 2 2 7 6 r + 49 36 = ( 1 r 7 6 ) 2 = 0 2(\frac{1}{4} + \frac{1}{9} + 1 + \frac{1}{r^2}) = (\frac{1}{2} + 1 - \frac{1}{3} + \frac{1}{r})^2 \Rightarrow \frac{49}{18} + 2\frac{1}{r^2} = (\frac{7}{6} + \frac{1}{r})^2 \Rightarrow \frac{1}{r^2} - \frac{2\cdot 7}{6r} + \frac{49}{36} = (\frac{1}{r} - \frac{7}{6})^2 = 0

q.e.d

0 pending reports

×

Problem Loading...

Note Loading...

Set Loading...